A DEF has vertices D(-5,-1), E (3, 3), and F (1,-5). Find the equation of the altitude through F in
slope-intercept form.

Answers

Answer 1

Answer:

y = -2x -3

Step-by-step explanation:

- the altitude trough F is a perpendicular line to the line DE

- find slope of line DE

D ( x2 = -5, y2 = -1); E (x1 = 3, y1 = 3)

slope m = (y2-y1) / (x2-x1) = (-1-3) / (-5-3) = -4/ -8 = 1/2

-find equation of the altitude trough F

lines that are perpendicular have the slope negative reciprocal (negative reciprocal of 1/2 is -2)

y= -2x +b , for point F(1, -5)

-5 = -2*1 +b, add 2 to both sides

-5 +2 = b, combine like terms

-3 =b

equation of the altitude trough F is y = -2x -3


Related Questions

Can someone help me with this math homework please!

Answers

Answer:

Second option

As the x-values go to negative infinity, the function´s values go to positive. infinity.

--------------------------

Hope it helps...

have a great day!!

Answer:

(B) As the x-values go to negative infinity, the function's values go to positive infinity.

Step-by-step explanation:

The x-values the answer choices are talking about are the values on the x-axis.

Looking at the graph, when the x-values go to negative infinity (meaning it keeps on going left, or negative), the function goes up, meaning the function goes to positive infinity.

When the x-values go to positive infinity (meaning it keeps on going right, or positive), the function goes up, meaning the function goes to positive infinity.

Out of all the answer choices, only B agrees with the observations written above.

Hope that helps (●'◡'●)

Find f(-3) for f(x) = 4(2)^x
O A. -32

O B. 1/2

O C. -24

O D. 1/8

Answers

Answer:

B. 1/2

General Formulas and Concepts:

Pre-Algebra

Order of Operations: BPEMDAS

Brackets Parenthesis Exponents Multiplication Division Addition Subtraction Left to Right

Algebra I

Functions

Function Notation

Step-by-step explanation:

Step 1: Define

Identify

f(x) = 4(2)ˣ

Step 2: Evaluate

Substitute in x [Function f(x)]:                                                                           f(-3) = 4(2)⁻³Exponents:                                                                                                        f(-3) = 4(1/8)Multiply:                                                                                                             f(-3) = 1/2

what is 3x10x178 I need help asap

Answers

Answer:

3x10=30   30x178=5,430

Step-by-step explanation:

QUESTION:- what is 3x10x178

ANSWER:-

[tex]3 \times 10 \times 178 \\ 30 \times 178 \\ 5340 \: \: answer[/tex]

Can someone help me with this math homework please!

Answers

Answer:

f(n+1) = f(n) - 5

Step-by-step explanation:

Just find some relationship between 2 numbers that are next to each other.

which of the following pairs of equations are perpendicular

Answers

Answer:

were are the pairs?

Which table of values could be generated by the equation 10x+5y=15? (Will give brainlest and 21 points)

Answers

Answer:

(For the image) A

Step-by-step explanation:

 The angle of elevation to a nearby tree from a point on the ground is measured to be 65°. How tall is the tree if the point on the ground is 92 feet from the tree? Round your answer to the nearest hundredth of a
foot if necessary.

Answers

Answer:

197.3 feet

Step-by-step explanation:

197.295 rounded to the nearest hundredth is 197.30 or 197.3

The height of the tree if the point on the ground from the tree is 92 feet will be 197.29 feet.

What is a right-angle triangle?

It is a type of triangle in which one angle is 90 degrees and it follows the Pythagoras theorem and we can use the trigonometry function. The Pythagoras is the sum of the square of two sides is equal to the square of the longest side.

The angle of elevation to a nearby tree from a point on the ground is measured to be 65°.

The height of the tree if the point on the ground from the tree is 92 feet. Then we have

Let h be the height of the tree. Then we have

[tex]\tan 65^o = \dfrac{h}{92}\\\\ 2.1445\ = \dfrac{h}{92}[/tex]

Then we have

[tex]\rm h = 2.1445 \times 92\\\\h = 197.29\ ft[/tex]

More about the right-angle triangle link is given below.

https://brainly.com/question/3770177

A car travels 600 km in 6 hours. at what rate of speed is the car traveling?

Answers

Step-by-step explanation:

100Km/hours......V/T

Write each power as a single power, then evaluate (9

Answers

Answer:

Step-by-step explanation:

a.  〖(9^8)〗^0 = 1

b.  〖〖[(-2)〗^4]〗^2 = 256

c.  〖–(3^2)〗^3 = -729

Find the length of side BC give your answer to three significant figures

Answers

Answer:

19.4 cm

Step-by-step explanation:

Hi there!

This is a right triangle. We're given an angle, the side adjacent to the angle and we're solving for the hypotenuse. Given this information, we can use the cosine ratio:

[tex]cos\theta=\frac{adj}{hyp}[/tex]

Plug in the given angle and side

[tex]cos71=\frac{6.3}{BC}\\BC=\frac{6.3}{cos71} \\BC=19.4[/tex]

Therefore, the length of BC is 19.4 cm when rounded to 3 significant figures.

I hope this helps!

Please HELP!! Will mark brainliest!!!

Answers

Answer:

x=6

y=5

Step-by-step explanation:

15x and 90 are vertical angles and vertical angles are equal

15x = 90

Divide by 15

15x/15 = 90/15

x = 6

15x and 5x+12y are corresponding angles and corresponding angles are equal when the lines are parallel

15x= 5x+12y

Subtract 5x

15x-5x = 5x+12y-5x

10x = 12y

10(6) = 12y

60 = 12y

Divide by 12

60/12 = 12y/12

5 = y

Isabella runs 3 times per week. She ran for 15 minutes on Monday and 17 minutes on Wednesday. Her coach told her that she had run 80 percent of her goal that week. How many more minutes does she need to run to meet her goal for the week?

Answers

Answer

8 More minutes

Step-by-step explanation:

15+ 17 = 32

32/.8 = 40

40 - 32 = 8

A triangle has an area of 18ft squared. List all the possible positive integers that could represent its base and height. Use sentences to describe your process.

Answers

Answer:

1ft x 36ft

2ft x 18ft

3ft x 12ft

4ft x 9ft

6ft x 6ft

Step-by-step explanation:

A triangle is a three-sided polygon with three edges and three vertices. the sum of angles in a triangle is 180 degrees

Area of a triangle = 1/2 x base x height

If area is 18ft², then the dimensions of base x height = 18 x 2 = 36

all the possible positive integers that could represent its base and height can be determined be known by determining the factors of 36

factors of 36 = 1, 2, 3, 4, 6, 9, 12, 18, and 36

Possible dimensions of base and heights are :

1ft x 36ft

2ft x 18ft

3ft x 12ft

4ft x 9ft

6ft x 6ft

The population of China is about 1.4 x 10^9 people. Julie grew up in a town that had a population of about 7,000 people. How many times greater is the population of China than Julie's hometown?


Please write your answer in standard notation

Answers

Answer:

200,000 times

Step-by-step explanation:

The population of China is about 1.4 × 10⁹ . And the hometown of Julie has a population of 7,000 people . We need to find out how many times greater is the population of China than Julie's hometown .

By Question :-

[tex]\rm\implies Population_{China}= 1.4\times 10^9 [/tex]

[tex]\rm\implies Population_{Hometown}= 7,000 [/tex]

Times by which population of China is more:-

For finding this divide the population of China by population of hometown .

[tex]\rm\implies \dfrac{ 1.4 \times 10^9 }{7000} \\\\\rm\implies \dfrac{ 14 \times 10^8}{7\times 10^3}\\\\\rm\implies 2\times 10^{8-3} \\\\\rm\implies 2\times 10^5 \\\\\rm\implies \boxed{\quad \bf 200,000 \quad }[/tex]

Therefore the population of China is greater than 200,000 times .

What is the range of f(x)=4^x

Answers

Answer:

B

Step-by-step explanation:

At - infinity, the function will tend to 0 and at +infinity, the function will tend to +infinity. Those are the two extremas of the function and extremas define the range. Range is all positive real number

Find the measure of the missing angle using the exterior angle sum theorm.

Answers

Answer:

95 degrees

Step-by-step explanation:

First find the angles within the triangle. 180 - 43 - 52 = 85, so the missing angle within the triangle is 85. The exterior angle would be supplementary to that, so 180 - 85 = 95.

Aisha wants to paint the four walls of her living room.
Each wall is 2.2 m high and 5.5 m long.
One wall has a door of 1.8 m by 0.9 m.
Tins of paint cost £13 per 2 L tin.
Each litre of paint can cover 8 m2 of wall.
There is an offer of: Buy 2 tins get the 3rd at half price.
How much will Aisha pay to paint her living room?

Answers

Answer:

£32.50

Step-by-step explanation:

my first question to the teacher : so, no windows in the living room ?

so, it is a square living room with 5.5 m side length.

but each wall is a rectangle of 2.2 × 5.5 m.

for one wall we have to deduct a door area of 1.8×0.9 m.

so, one wall

2.2 × 5.5 = 12.1 m²

4 walls

4 × 12.1 = 48.4 m²

minus one door area

1.8 × 0.9 = 1.62 m²

48.4 - 1.62 = 46.78 m² total paint area

1 L paint covers 8 m².

so, we need 46.78/8 = 5.85 liters.

she gets the paint in 2 L tins. so, she needs 3 tins (6 L).

each tin costs £13.

and because she buys 3 tins, she gets the third one for half the price (13/2 = £6.50).

so, she has to pay

2×13 + 6.50 = 26 + 6.50 = £32.50

I am having trouble with this problem how do i solve it.

Answers

Answer:

90

Step-by-step explanation:

I do not know if this is true or not but i am pretty sure this is perimeter

All triangles sides have to add up to 180

1. add up 25 and 65

2. 180-90=90

Then i think your answer is 90

Please do not give me hate if i did this wrong


use a double angle or half angle identity to find the exact value of each expression

Answers

Answer:

Step-by-step explanation:

There are 2 very distinct and important things that we need to know before completing the problem. First is that we are given that the cos of an angle is 1/3 (adjacent/hypotenuse) and it is in the first quadrant. We also need to know that the identity for sin2θ = 2sinθcosθ.

We already know cos θ = 1/3, so we need now find the sin θ. The sin ratio is the side opposite the angle over the hypotenuse, and the side we are missing is the side opposite the angle (we do not need to know the angle; it's irrelevant). Set up a right triangle in the first quadrant and label the base with a 1 (because the base is the side adjacent to the angle), and the hypotenuse with a 3. Find the third side using Pythagorean's Theorem:

[tex]3^2=1^2+y^2[/tex] which simplifies to

[tex]9=1+y^2[/tex] and

[tex]y^2=8[/tex] so

[tex]y=\sqrt{8}=2\sqrt{2}[/tex] so that's the missing side. Now we can easily determine that

[tex]sin\theta=\frac{2\sqrt{2} }{3}[/tex]

Now we have everything we need to fill in the identity for sin2θ:

[tex]2sin\theta cos\theta=2(\frac{2\sqrt{2} }{3})(\frac{1}{3})[/tex] and multiply all of that together to get

[tex]2sin\theta cos\theta=\frac{4\sqrt{2} }{9}[/tex]

please help me its urgent​

Answers

Answer:

number of students who like only iphone is 36...

Step-by-step explanation:

100 is the total number..

from that 40 like both so we subtract..

100-40=60..

60 = 2x + 3x..

60 / 5 = 12..

so numbet who like iphone is 3x which is 3 * 12=36..

Grandma is making a quilt. She has 540 cm of fabric to border the quilt. What is the greatest possible area for the quilt?
Question 1 options:


11 664 cm^2


18225 cm^2


72900 cm^2


291600 cm^2

Show your work:

Answers

Answer:

18225 cm²

Step-by-step explanation:

Divide 540 by 4 to get the length of all sides

540/4 = 135

Square 135 to get the max possible size

135² = 18225

18225 cm²  is the greatest possible area for the quilt.

What is area?

The measurement that expresses the size of a region on a plane or curved surface is called area. Surface area refers to the area of an open surface or the boundary of a three-dimensional object, whereas the area of a plane region or plane area refers to the area of a form or planar lamina.

Given

Divide 540 by 4 to obtain the length of all sides

540/4 = 135

Square 135 to acquire the max possible size

135² = 18225

18225 cm²  is the greatest possible area for the quilt.        

To learn more about area refer to:

https://brainly.com/question/25292087

#SPJ2

Three red balls, 5 green balls and a number of blue balls are put together in a sac. One ball is picked at random from the sac. If the probability of picking a red ball is 1|6, find the a) The number of blue balls in sac. B) the probability of picking a green ball​

Answers

Answer:

total balls = 18 .... 3/x = 1/6

blue = 10 ... 18-(5+3) = 10

p of green = 5/18 = .277

Step-by-step explanation:

What is the solution to this system of linear equations?
2%. + 3y = 3
7x-3y = 24
O (
27)
(3,-21)
O (3-1)
19.0

Answers

Answer:

(3,-1)

Step-by-step explanation:

2x + 3y = 3  -----------(I)

7x - 3y = 24 ----------(II)

Add equation (I) & (II) and y will be eliminated and we can find the value of 'x'

(I)         2x + 3y = 3

(II)       7x   - 3y = 24   {Add}

         9x           = 27   {Divide both sides by 9}

                    x = 27/9

x = 3

Plugin x = 3 in equation (I)

2*3 + 3y = 3

  6 + 3y = 3

Subtract 6 from both sides

 3y = 3 - 6

 3y = -3

Divide both sides by 3

    y = -3/3

y = -1

   

Convert to decimal degrees.

-(167° 31”)

[?]°

Enter your answer with three decimal places.

Answers

Answer:

The angle in decimal form is 167.009°.

Step-by-step explanation:

We know an angle in terms of integer angles, minutes and seconds, whose conversion into decimal degrees is expressed by the following formula:

[tex]\theta = n + \frac{m}{60}+\frac{s}{3600}[/tex] (1)

Donde:

[tex]n[/tex] - Integer angle, in sexagesimal degrees.

[tex]m[/tex] - Minutes.

[tex]s[/tex] - Seconds.

If we know that [tex]n = 167[/tex], [tex]m = 0[/tex] and [tex]s = 31''[/tex], then the angle in decimal form is:

[tex]\theta = 167^{\circ}+\frac{0}{60}^{\circ} + \frac{31}{3600}^{\circ}[/tex]

[tex]\theta = 167.009^{\circ}[/tex]

The angle in decimal form is 167.009°.

Find the length of the segment indicated. Round your answer to the nearest 10th if necessary.

Answers

Answer:

x=13.6

Step-by-step explanation:

By Pythagoras theorem, 5.5^2+x^2=14.7^2. x^2=14.7^2-5.5^2. x=13.6

Find cosθ+cos3θ+cos5θ+cos7θ by using the Sum-to-Product Formula.
Please also show your work as well. Thanks!

Answers

Answer:

[tex] \rm\displaystyle 4\cos( \theta) \cos \left( {2\theta} \right) \cos \left( {4 \theta } \right) [/tex]

Step-by-step explanation:

I assume the question want us to rewrite cosθ+cos3θ+cos5θ+cos7θ by using Sum-to-Product Formula and note that it's not an equation therefore θ can never be specified

===========================

so we want to rewrite cosθ+cos3θ+cos5θ+cos7θ by using Sum-to-Product Formula the good news is that the number of the function of the given expression is even so there's a way to do so, rewrite the expression in parentheses notation:

[tex] \rm\displaystyle \left( \cos( \theta) + \cos(3 \theta) \right) + \left(\cos(5 \theta) + \cos(7 \theta) \right)[/tex]

recall that,Sum-to-Product Formula of cos function:

[tex] \rm \boxed{\displaystyle \cos( \alpha ) + \cos( \beta ) = 2 \cos \left( \frac{ \alpha + \beta }{2} \right) \cos \left( \frac{ \alpha - \beta }{2} \right) }[/tex]

notice that we have two pair of function with which we can apply the formula thus do so,

[tex] \rm\displaystyle \left( 2\cos \left( \frac{ \theta + 3 \theta}{2} \right)\cos \left( \frac{ \theta - 3 \theta}{2} \right) \right) + \left(2\cos \left( \frac{5 \theta + 7 \theta}{2} \right) \cos \left( \frac{5 \theta - 7 \theta}{2} \right) \right)[/tex]

simplify addition:

[tex] \rm\displaystyle \left( 2\cos \left( \frac{4 \theta}{2} \right)\cos \left( \frac{ - 2\theta }{2} \right) \right) + \left(2\cos \left( \frac{12 \theta }{2} \right) \cos \left( \frac{ - 2 \theta}{2} \right) \right)[/tex]

simplify division:

[tex] \rm\displaystyle \left( 2\cos \left( {2 \theta} \right)\cos \left( { - \theta } \right) \right) + \left(2\cos \left( {6 \theta } \right) \cos \left( { - \theta} \right) \right)[/tex]

By Opposite Angle Identities we acquire:

[tex] \rm\displaystyle \left( 2\cos \left( {2 \theta} \right)\cos \left( { \theta } \right) \right) + \left(2\cos \left( {6 \theta } \right) \cos \left( { \theta} \right) \right)[/tex]

factor out 2cosθ:

[tex] \rm\displaystyle 2 \cos( \theta) (\cos \left( {2 \theta} \right) + \cos \left( {6 \theta } \right) )[/tex]

once again apply Sum-to-Product Formula which yields:

[tex] \rm\displaystyle 2 \cos( \theta) (2\cos \left( {4\theta} \right) \cos \left( {2 \theta } \right) )[/tex]

distribute:

[tex] \rm\displaystyle 4\cos( \theta) \cos \left( {2\theta} \right) \cos \left( {4 \theta } \right) [/tex]

and we're done!

please help me with this problem !

Answers

Answer: 2/3

Step-by-step explanation:

1/4 (3/12) plus 5/12 = 8/12

which can be simplified by 4

Find cos 0
A. 15/8
B. 15/17
C. 8/15
D. 8/17

Answers

Answer:

A.15/8

Step-by-step explanation:

the answer is 15/8

Answer:

D.

[tex]{ \tt{ \cos( \theta) = \frac{adjacent}{hypotenuse} }} \\ \\ { \tt{ \cos( \theta) = \frac{8}{ \sqrt{ {15}^{2} + {8}^{2} } } }} \\ \\ { \tt{ \cos( \theta) = \frac{8}{ \sqrt{289} } }} \\ \\ { \tt{ \cos( \theta) = \frac{8}{17} }}[/tex]

If f is continuous for all x, which of the following integrals necessarily have the same value?

Answers

Answer:

B

Step-by-step explanation:

Given the three integrals, we want to determine which integrals necessarily have the same value.

We can let the first integral be itself.

For the second integral, we can perform a u-substitution. Let u = x + a. Then:

[tex]\displaystyle du = dx[/tex]

Changing our limits of integration:

[tex]u_1=(0)+a=a \text{ and } u_2 = (b+a)+a = b+2a[/tex]

Thus, the second integral becomes:

[tex]\displaystyle \int_{0}^{b+a}f(x+a)\, dx = \int_a^{b+2a} f(u)\, du[/tex]

For the third integral, we can also perform a u-substitution. Let u = x + c. Then:

[tex]\displaystyle du = dx[/tex]

And changing our limits of integration:

[tex]\displaystyle u_1=(a-c)+c=a \text{ and } u_2=(b-c)+c=b[/tex]

Thus, our third integral becomes:

[tex]\displaystyle \int_{a-c}^{b-c}f(x+c)\, dx = \int_{a}^{b} f(u)\, du[/tex]

Since the only difference between f(x) and f(u) is the variable and both the first and third integral have the same limits of integration, our answer is B.

A square has a side length of 36 feet. This square is dilated by a scale factor of 2/3 to create a new square. What is the side length of the new square?

Answers

Answer:

24

Step-by-step explanation:

Multiply the side length by the dilation

36 x 2/3

72/3

Simplify

72/3 = 24

Your answer is correct

Other Questions
Find the measure of side C and round to the nearest whole number. Pls help ASAP 10 points please help im confusedWhat is the volume of the prism?103.8 cubic inches114.9 cubic inches140.6 cubic inches110.5 cubic inches A physical pendulum in the form of a planar object moves in simple harmonic motion with a frequency of 0.680 Hz. The pendulum has a mass of 2.00 kg, and the pivot is located 0.340 m from the center of mass. Determine the moment of inertia of the pendulum about the pivot point. Carmen MartinezWhat is the slope of the line that passes through the point 4,4 and 10,7 write your answer in simplest form What is the meaning of freedom ? ( in big terms, very descriptive) Lusine saw birds and mice eating seeds on the ground. She calculated 12 heads and 39 legs in total. How many birds and mice are on the ground? one similarity between physics and chemistry is... slove the equation for y and x The freezing point of oxygen is 218.790C and hydrogen is 252.80C. A lab is lowering the temperature inside a fridge. Which freezes first? How much colder does it have to be for both to freeze? Can A KIND SOUL HELP ME OUT??? ALL THE ANSWERS I SEE ARE WRONG FOR THIS PROBLEM!!!!!!!!!!:(((Find the measures of angles x, y and z in the figure.please show how you got it so I can see if the answer makes sense!!!! which statement accuretly contrasts the effects of judahism and christianity? Hydrogen chloride decomposes to form hydrogen and chlorine, like this: 2HCl(g) + H2(g) Cl2(g) Also, a chemist finds that at a certain temperature the equilibrium mixture of hydrogen chloride, hydrogen, and chlorine has the following composition: compound pressure at equilibrium HCl 84.4 atmH2 77.9 atmCl2 54.4Required:Calculate the value of the equilibrium constant for this reaction. Round your answer to significant digits. The HCF of 85 and 95 What's the angle sum of a polygon having 7 sides. Research indicates that television advertisements show Which statement best explains Douglass's purpose for writing this passage?to explain why it was important for him to learn to read and writeto illustrate how kind his mistress was and how terrible his master wasto share his thoughts on having young children teach him to readto address the evils of slavery and the need for freedom Leo carefully pipets 50.0 mL of 0.500 M NaOH into a test tube. She places the test tubeinto a small beaker to keep it from spilling and then pipets 75.0 mL of 0.250 M HCl intoanother test tube. When Leo reaches to put this test tube of acid into the beaker alongwith test tube of base she accidentally knocks the test tubes together hard enough tobreak them and their respective contents combine in the bottom of the beaker. Is thesolution formed from the contents of the two test tubes acidic or basic? What is the pH ofthe resulting solution?Please answer below questions one by one to assist you receive full credits(Alternatively, you can discard my hints below, solve the problem using your own wayand send me the picture/copy of your complete work through email)The mole of NaOH before mixing ismol (save 3 significant figures)The mole of HCl before mixing ismol (save 4 significant figures)After mixing, the solution is(choose from acidic or basic)The total volume of mixture isL (save 3 significant figures)The concentration of [OH-] isM (save 3 significant figures)The concentration of [H'l isM (save 3 significant figures) What is the least common multiple of 46, 49, and 17? 1 =1 ? herty rta4356gb? is g sdfgbns iesx? The rate of earnings is 6% and the cash to be received in 4 years is $20,000. The present value amount, using the following partial table of presentvalue of $1 at compound interest isYear6%10%12%10.9430.9090.89320.8900.8260.79730.8400.7510.71240.7920.6830.636a. $12.720Ob. $16,800Oc. 513,660Od. $15.840